LSAT and Law School Admissions Forum

Get expert LSAT preparation and law school admissions advice from PowerScore Test Preparation.

 moalkhaf
  • Posts: 10
  • Joined: Aug 30, 2017
|
#40111
Good afternoon. I was hoping that someone could clarify the fourth Grouping and Linear combination Drill question. The first question is a must be true question that places D on the West stage and N on Saturday. The correct answer is that F preforms on Thursday but can't K also preform on Thursday. If so, wouldn't that make the proposed answer a could be true instead of a must be true.
User avatar
 Dave Killoran
PowerScore Staff
  • PowerScore Staff
  • Posts: 5852
  • Joined: Mar 25, 2011
|
#40115
Hi M,

As a starter, here are two prior discussions of this particular drill item that you might find helpful (just in case):

More specifically to your question, we know from the setup that only D, F, K, and L can possibly perform on Thursday on the East stage. The question stem schedules D to perform on the west stage, leaving only F, K, and L for Thursday on the East stage. From the 4th rule, we can also rule K out from performing on Thursday on the East stage, leaving only F and L as possibilities for Thursday's East stage performance.

However, the question stem also schedules N for Saturday, and because of the effects of the first rule, this means that L must perform on Friday. With L now removed from Thursday, that leaves only F remaining to perform on Thursday on the East stage, and thus answer choice (B) must be true and is correct.

You mention K performing on Thursday; I think you meant K performing on Thursday on the East stage, but that is not possible since D must perform on Thursday per the question stem, and the 4th rule states that "D and K cannot perform on the same day."

Please let me know if that helps. Thanks!
 moalkhaf
  • Posts: 10
  • Joined: Aug 30, 2017
|
#40117
Oh I see. I forgot about the fourth rule. I appreciate the timely response.
 Hari
  • Posts: 2
  • Joined: Jan 11, 2018
|
#42809
Hello, I'm new to the forum. The Bible has been an excellent resource, thank you for providing it!

I had a question about the stimulus on one of the questions.

I'm on the Kindle version, the question appears at location 6512 of 11988. Question 4.1 of the setup drill in the grouping/linear combination chapter.

This is a Local, must be true question in an overloaded scenario. Part of the challenge in this setup was not explicitly knowing if comics could appear more than once. The explanation of the answer to Q4.1 seems to make it clear that they cannot.

My question is if this was an oversight, or if this question was taken directly from a previous LSAT? In either case, are we to assume (unless otherwise instructed) that variables cannot be placed more than once in overloaded scenarios?

Please advise, thanks!
User avatar
 Dave Killoran
PowerScore Staff
  • PowerScore Staff
  • Posts: 5852
  • Joined: Mar 25, 2011
|
#42843
Hi Hari,

Thanks for the question! The key here is the language in the first sentence: "From ten comics...exactly eight will be scheduled for individual performances." That is meant to tell you that 8 of the 10 comics are used, and since there are only 8 performances, each comic performs just once.

Please let me know if that helps. Thanks!
 Hari
  • Posts: 2
  • Joined: Jan 11, 2018
|
#42890
Hi Dave,

That clarifies it, thank you!

Hari

Get the most out of your LSAT Prep Plus subscription.

Analyze and track your performance with our Testing and Analytics Package.